具体数学第六章习题选做(genshining)

11.对于 \(n\ge 0\),求以下式子的封闭形式。

\[\sum_k(-1)^k{n\brack k} \]

由于

\[\sum{n\brack k}x^k=x^{\overline n} \]

原式即等于 \((-1)^{\overline n}=[n=0]\)

12.证明斯特林反演。代入即可。

13.证明:

\[\vartheta^n=\sum_k{n\brace k}z^kD^k\\z^nD^n=\sum_k {n\brack k}(-1)^{n-k}\vartheta^k \]

归纳证明上式。

\[\vartheta\circ(\sum_k{n-1\brace k}z^kD^k)\\ =\sum_k{n-1\brace k}z(kz^{k-1}D^k+z^kD^{k+1})\\ =\sum_k\left({n-1\brace k}k+{n-1\brace k-1}\right)z^kD^k\\ =\sum_k{n\brace k}z^kD^k \]

边界条件显然成立。

对上式斯特林反演即得到下式。

注记:我们在什么时候用到呢?想想超几何函数的微分方程。

14.证明 Worpitzky 恒等式。

\[x^n=\sum_k\left\langle \begin{matrix} n\\k \end{matrix} \right\rangle\binom{x+k}{n} \]

容易得知:

\[x\binom{x+k}{n}=(k+1)\binom{x+k}{n+1}+(n-k)\binom{x+k+1}{n+1} \]

然后发现拆开右式的

\[\binom{x+k}{n} \]

就可以归纳证明。

15.证明

\[m!{n\brace m}=\sum _k\left\langle \begin{matrix} n\\k \end{matrix} \right\rangle\binom{k}{n-m} \]

对 Worpitzky 恒等式两边求 \(x\)\(m\) 阶有限微积分得到:

\[\Delta^m(x^n)\mid_{x=0}=\Delta^m\left(\sum_k\left\langle \begin{matrix} n\\k \end{matrix} \right\rangle\binom{x+k}{n}\right)\mid_{x=0}\\ \sum_i \binom{m}{i}i^n(-1)^{m-i}=\sum_k\left\langle \begin{matrix} n\\k \end{matrix} \right\rangle\binom{k}{n-m}\\ \sum_k\left\langle \begin{matrix} n\\k \end{matrix} \right\rangle\binom{k}{n-m}=m!{n\brace m}\\ \]

解递归式:

\[A_{n,0}=a_n[n\ge 0];A_{0,k}=0,k>0;\\ A_{n,k}=kA_{n-1,k}+A_{n-1,k-1} \]

只给答案是什么意思/yiw

\(F_n(x)=\sum A_{n,i}x^i\)

可以发现,

\[F_n(x)=xDF_{n-1}(x)+xF_{n-1}(x)+a_n\\ =(\vartheta+x) F_{n-1}+a_n\\ =\sum (\vartheta+x)^{n-i}a_i\\ \]

注意到这里的算子代数乘法是复合,不满足一些运算律。

可以归纳证明

\[(\vartheta+x)^n(C)=C\sum_i {n\brace i}x^i \]

那么原式等于

\[\sum_i a_i\sum _j {n-i\brace j}x^j\\ =\sum _jx^j\sum _i a_i{n-i\brace j}\\ =\sum _ix^i\sum _j a_j{n-j\brace i} \]

所以 \(A_{n,i}=\sum _j a_j{n-j\brace i}\)

19.证明:

\[\sum _{k=0}^n{x+k\brace x}{x\brack x-n+k}(-1)^k/\binom{x+k}{n+1}\\ =\sum _{k=0}^n{-x\brack -x-k}{x\brack x-n+k}(-1)^k/\binom{x+k}{n+1}\\ =\sum _{k=0}^n\sigma _{k}(-x)(-x)^{\underline{k+1}}\sigma_{n-k}(x)x^{\underline {n-k+1}}(-1)^k/\binom{x+k}{n+1}\\ =-(n+1)!x\sum_{k=0}^n \sigma_k(-x)\sigma_{n-k}(x) \]

而我们知道

\[rs\sum_k\sigma_k(r+tk)\sigma_{n-k}(s+t(n-k))=(r+s)\sigma_n(r+s+tn) \]

\(t=0,r=-x,s=x\) 即可发现原式为 \(0\)

21.没看懂

22.证明:除了 \(z\in (-\infty,0)\cap \Z\)

\[\sum_{k\ge 1}(\frac{1}{k}-\frac{1}{k+z}) \]

\(z\in \C\) 收敛,且 \(z\in\N\) 时,原式 \(=H_x\)

后面一个是容易的。前面一个可以放缩:当 \(k>2|z|\) 时,

\[\frac1k-\frac1{k+|z|}\le\frac{2|z|}{k^2} \]

\[\sum _{k\ge 1}\frac{2|z|}{k^2} \]

收敛。

23.求

\[[\frac{z^m}{m!}]\frac{z}{e^z+1} \]

首先知道

\[\frac{z}{e^z-1}=\sum_{k\ge 0}B_k\frac{z^k}{k!} \]

\[\frac{z}{e^z+1}=\frac{z(e^z-1)}{e^{2z}-1}\\ =\frac{ze^z}{e^{2z}-1}-\frac{z}{e^{2z}-1}\\ =\frac{z}{e^{z}-1}+\frac{z}{e^{2z}-1}-\frac{2z}{e^{2z}-1}\\ =\frac{z}{e^z-1}-\frac{2z}{e^{2z}-1}\\ =\sum_{k\ge 0}(1-2^k)B_k\frac{z^k}{k!} \]

24.证明:\(2^n\mid T_{2n+1}\),其中 \(T_n\) 是正切数。

考虑 \(T_{2n+1}(x)\) 的系数,一定只在偶数次项有值。

\(T_{n}=(1+x^2)T_{n-1}'(x)\),求导两次和乘 \((1+x^2)\) 之后系数至少乘二,而仍然只有偶数项有值。

26.计算

\[C=\sum_1^{n+1} \frac{H_k}{k}\delta k\\ =\sum_1^{n+1} H_k\delta H_{k-1}\\ =H_nH_{n+1}-\sum _1^{n+1}\frac{H_k}{k+1}\delta k\\ =H_nH_{n+1}-\sum _2^{n+2}\frac{H_{k}}{k}\delta k-\sum _2^{n+2}\frac{1}{k^2}\delta k\\ =H_nH_{n+1}+1-\frac{H_{n+1}}{n+1}-C+1-H_{n+1}^{(2)}\\ 2C=H_nH_{n+1}+2-\frac{H_{n+1}}{n+1}-H_{n+1}^{(2)}\\ C=\frac{H_nH_{n+1}-H_{n+1}^{(2)}+2}{2}-\frac{H_{n+1}}{2(n+1)} \]

27.证明:

\[(F_m,F_n)=F_{(m,n)} \]

\(m>n\)

\[(F_m,F_n)=(F_{m-n}F_{n+1}+F_{m-n-1}F_n,F_n)\\ =(F_{m-n}F_{n+1},F_n) \]

\((F_n,F_{n+1})\) 容易证明是 \(1\),则 \((F_m,F_n)=(F_{m-n},F_n)\)。归纳证明。

\(L_n=F_{n+1}+F_{n-1}\)

\(F_n,L_n\) 表示 \(Q_n\),其中 \(Q_n=Q_{n-1}+Q_{n-2},Q_0=\alpha,Q_1=\beta\)

容易使用矩阵得到 \(Q_n=\alpha F_{n-1}+\beta F_n\),用 \(L_n\) 是容易的。

29.30.等我学了连项式再说

证明:\(\forall \epsilon\in (0,1),\exists n>1,H_n\bmod1<\epsilon\)

\(n>\dfrac{1}{\epsilon}\),且 \(\lfloor H_n\rfloor>\lfloor H_{n-1}\rfloor\),根据 \(H_n\) 的发散性,这样的 \(H_n\) 一定存在;\(H_n\) 即为答案。

38.求

\[\sum_{k=1}^n\binom rk(-1)^kH_k \]

的封闭形式。

\[\sum_{k=1}^n\binom rk(-1)^kH_k\\ =\sum_{k=1}^n\binom rk(-1)^k\sum_{j=1}^k\frac 1j\\ =\sum _{j=1}^n\frac 1j\sum_{k=j}^n\binom rk(-1)^k\\ =\sum_{j=1}^n\frac{(-1)^n\binom{r-1}{n}+(-1)^j\binom{r-1}{j-1}}{j}\\ =H_n(-1)^n\binom{r-1}n+\frac1r\sum_{j=1}^n(-1)^j\binom{r}{j}\\ =H_n(-1)^n\binom{r-1}n-\frac 1r \]

39.求

\[\sum H_k^2\delta k \]

考虑 \(\delta (H_k^3)\) 和之前已经得到的结果即可。

\[=(-1)^k\left(\binom{r-1}kr^{-1}-\binom{r-1}{k-1}H_k\right) \]

42.设 \(S\subset \Z\)\(S+1=\{x+1\mid x\in S\}\)

\(S\subseteq \{1,2,\dots,n\}\)\(S\cup (S+1)=\{1,2,\dots,n+1\}\)\(S\) 个数。

一种有趣的做法:考虑 \(a_i=[i\in S]\),可以使用动态规划得出其递推式:把递推式写为线性变换形式,可以化为矩阵幂,发现

\(\operatorname{Card}(S)=F_n\)

43.求

\[0.1\\ +0.01\\ +0.002\\ +0.0003\\ +0.00005\\ \dots \]

的值。

考虑

\[\sum_{n\ge 0} F_nx^n=\frac{1}{1-x-x^2} \]

\(x=10^{-1}\),不难发现此式收敛,代入得到 \(=\dfrac{10}{89}\)

47.证明:

\[2^{n-1}F_n=\sum _k\binom n{2k+1}5^k \]

并求出 \(F_p\bmod p\)\(F_{p+1}\bmod p\)

这里给出 GF 解法:

\[\sum_{n\ge 0} F_n2^{n-1}z^n=\frac 12\sum_{n\ge 0} F_n(2z)^n\\ =\frac{2z}{2-4z-8z^2}=\frac{z}{1-2z-4z^2}\\ \sum_{n\ge 0}z^n\sum_k\binom{n}{2k+1}5^k\\ =\sum_k5^k\sum_{n}\binom n{2k+1}z^n\\ =\sum _k5^k\frac{z^{2k+1}}{(1-z)^{2k+2}}\\ =\frac{z}{(1-z)^2}\sum _{k\ge 0}\left(\frac{5z^2}{(1-z)^2}\right)^k=\frac{z}{(1-z)^2}\frac{1}{1-\dfrac{5z^2}{(1-z)^2}}\\ =\frac{1}{1-2z-4z^2} \]

\(F_p\bmod p=5^{\frac{p-1}2},2F_{p+1}\bmod p=1+5^{\frac{p-1}{2}}\),这是不难的。

51.设 \(p\in \mathcal P\)

证明(Wolstenholme 定理):

\[(a)\forall 1<k<p,{p\brace k}\equiv {p\brack k}\equiv 0\pmod p\\ (b)\forall 1\le k<p,{p-1\brack k}\equiv 1\pmod p\\ (c)p>2\Rightarrow {2p-2\brace p}\equiv {2p-2\brack p}\equiv 0\pmod p\\ (d)p>3\Rightarrow {p\brack 2}\equiv 0\pmod {p^2} \]

(a)🐨 考虑:

\[x^p-x\equiv x^{\underline p}\pmod p \]

这是由于费马小定理。接下来展开右侧即可。

但是存在一个优美的组合证明。

考察在群(模意义下每个元素加 \(v,0\le v<p\))作用下的分成 \(k\) 个轮换(集合)的所有方案的集合。

根据轨道-稳定子定理,

\[St(x_0)\cdot |G(x_0)|=|G|=p \]

\(St(x_0)\)\(1<k<p\) 时总为 \(1\)\(v=0\)),故轨道 \(|G(x_0)|=p\)。那么由于原集合可以被分为若干不交轨道,所以 \(p\mid |\Omega|\)

(b)根据递推公式和威尔逊定理可以归纳 \((a)\Rightarrow (b)\)

(c)递推一下发现 \((a)\) 的上指标增大,范围两边缩小一。

(d)🐨 考虑:

\[p^{\underline p}=p!=\sum _{i}{p\brack i}(-1)^{p-i}p^i\\ \sum _{i>1}{p\brack i}(-1)^{p-i}p^i=0\\ {p\brack 2}=\sum _{i>2}{p\brack i}p^{i-2}(-1)^{p-i+1} \]

右式每一项都整除 \(p^2\)

52.设 \(H_n=a_n/b_n,\gcd(a_n,b_n)=1\)

(a)证明:\(p\in \mathcal P\Rightarrow p\mid b_n\iff p\not\mid a_{\lfloor n/p\rfloor}\)

(b)找出所有 \(n>0\),使得 \(5\mid a_n\)

(a):注意到:

\[H_n=\sum _{i=1}^n\frac{[(i,p)=1]}{i}+H_{\lfloor n/p\rfloor}/p \]

然后显然。

(b)不会。

53.求

\[\sum _{k=0}^m\binom nk^{-1}(-1)^kH_k \]

\[\sum _{k=0}^m\binom nk^{-1}(-1)^kH_k\\=\sum _{k=0}^m\binom nk^{-1}(-1)^k\sum_{j=1}^k \frac 1j\\ =\sum _{j=1}^m\frac 1j \sum _{k=j}^{m}\binom{n}k^{-1}(-1)^k \]

此式后式是第五章第 42 题(恰好没做,,,),是可以 Gosper 的。然后不难发现后式搞出来之后提一下 \(H_m\),原式又是可以 Gosper 的。(Gosper,yyds!)

54.设 \(n>0\)

(a)设 \(p\in \mathcal P,m>0\),那么:\(p\mid (S_m(p)+[(p-1)\mid m])\)

(b)证明:

\[I_{2n}=\left(B_{2n}+\sum_{p\in \mathcal P}\frac{[(p-1)\mid 2n]}{p}\right)\in \Z \]

(c)证明:\(B_{2n}\) 的分母总是 \(6\) 的倍数,且有无穷个 \(n\) 使其 \(=6\)

证明:

(a)

首先 \(S_m(p)=S_{m-(p-1)}(p),m\ge p-1\),由费马小定理。那么下面假设 \(m<p-1\)

接下来把 \(S_m\) 用斯特林展开幂即可。

(b)

只需证明:\(B_{2n}+\dfrac{(p-1)\mid 2n}{p}\) 的分母不被 \(p\) 整除。

考虑 (a) 中使用伯努利公式,结合 \(B_{2n-1}=0\),得到:

\[\left(B_{2n}+\frac{[(p-1)\mid 2n]}p+\sum _{k=0}^{2n-2}\binom{2n+1}kB_k\frac{p^{2n-k}}{2n+1}\right)\in \Z \]

只需证明

\[\binom{2n+1}kB_k\frac{p^{2n-k}}{2n+1}=\binom{2n}kB_k\frac{p^{2n-k}}{2n-k+1} \]

分母不含 \(p\)

可以归纳证明(我没证出来)\(B_k\) 的分母不整除 \(p^2\),而 \(p^{2n-k-1}/(2n-k+1)\) 分母不含 \(p\),是因为显然 \(V_p(n)\le n-2\),对于 \(n\ge 3\)

(c)

\[I_{2n}-\sum_{p\in \mathcal P}\frac{[(p-1)\mid 2n]}{p}=B_{2n} \]

\(p=2,3\) 时总有 \((p-1)\mid 2n\)。所以 \(B_{2n}\) 的分母总是 \(6\) 的倍数。

而对于 \(n\in\mathcal P,2n+1\not\in \mathcal P\),分母是 \(6\)。而这样的数是无限多的。因为考虑 \(n\mid 2^{n-1}n+2^{n-1}-1\),取 \(n=2n+1\) 不会一直是质数。所以有无限多。

56.计算以下式子的封闭形式:

\[\sum_{k\neq m}\binom nk (-1)^kk^{n+1}/(k-m) \]

\[\sum_{k\neq m}\binom nk (-1)^kk^{n+1}/(k-m)\\ =\sum_{k\neq m}\binom nk(-1)^k((k-m)+m)^{n+1}/(k-m)\\ =\sum_{k\neq m}\binom nk(-1)^k\sum _{j}\binom{n+1}j(k-m)^{j-1}m^{n+1-j}\\\\ =\sum _{j}\binom{n+1}jm^{n+1-j}\sum_{k}\binom nk(-1)^k(k-m)^{j-1}\\ \]

发现内式有值当且仅当 \(j=0,n+1\),因为这是 \(n\) 阶差分的形式。进而 \(j=n+1\) 的时候是 \((-1)^nn!\)

只需考虑 \(j=0\)

\[\sum_k\binom nk (-1)^k(k-m)^{-1} \]

现在仅考虑 \(m\in [0,n]\),否则根据 \((5.41)\)(qsy 的学号,考虑 \(f(x)=(x-1)^{\underline{-1}}\) 的差分),这是平凡的。

否则,🐨 考虑对

\[\sum_k\binom nk\frac{(-1)^k}{x+k}=x^{-1}\binom{x+n}n^{-1},x\not\in \{0,-1,\dots,-n\}.(5.41) \]

试图令 \(x=-m\),但不可行;所以计算

\[\lim _{x\to -m}x^{-1}\binom{x+n}{n}^{-1}-\binom nm\frac{(-1)^m}{x+m} \]

我暂时没极限出来,但是答案是:

\[(-1)^{m+1}\binom nm m^n (n+1+mH_{n-m}-mH_m) \]

这个调和数显然是对组合数求导来的,但是为什么我洛出来是 \(0\)(洛必错了)

60.求所有 \(F_n-1\)\(F_n+1\) 是质数的。

验证 \(\le 6\) 的。如果更大的,有:

\[F_{2m}+(-1)^m=L_{m+1}F_{m-1}\\ F_{2m+1}+(-1)^m=L_{m}F_{m+1}\\ F_{2m}-(-1)^m=L_{m-1}F_{m+1}\\ F_{2m+1}-(-1)^m=L_{m+1}F_{m}\\ \]

求以下式子的封闭形式

\[\sum_{k=0}^n \frac{1}{F_{2^k}} \]

考虑

\[F_{2m}=\frac{F_{m-1}}{F_m}-\frac{F_{2m-1}}{F_{2m}}\\ \sum_{k=0}^n \frac{1}{F_{2^k}}=2+\sum_{k=2}^n \frac{1}{F_{2^k}}\\ =2+\sum_{k=2}^n \frac{F_{2^{k-1}-1}}{F_{2^{k-1}}}-\frac{F_{2^{k}-1}}{F_{2^{k}}}\\ =2+\sum_{k=1}^{n-1}\frac{F_{2^{k}-1}}{F_{2^{k}}}-\sum_{k=2}^n\frac{F_{2^{k}-1}}{F_{2^{k}}}\\ =3-\frac{F_{2^n-1}}{F_{2^n}} \]

求有多少 \(n\) 阶排列 \(\pi_{1:n}\) 使得:

(a)有 \(k\) 个前缀最大值;

(b)有 \(k\) 个超过数: \(\pi_i>i\)

均考虑增量,列出 dp 方程,发现其等于某特殊数。

(a)的增量是 trival 的,直接考虑每次插入最小数即可。显然是 \({n\brack k}\)

(b)的增量需要一些技巧:考虑排列 \(\pi_{1:n}\) 插入 \(n+1\),以这样的方式插入:

\[(1)\pi_1\pi_2\dots\pi_n(n+1)\\ (2)\pi _1\pi_2\dots \pi_{j-1}(n+1)\pi_{j+1}\dots \pi _n\pi _j \]

第一种情况不会改变超过数。

第二种情况,如果 \(j\) 是超过数,那么没有改变超过数个数;否则增加了一个超过数。

列出 dp 方程发现是欧拉数 \(\left\langle\begin{matrix}n\\k\end{matrix}\right\rangle\)

64.求 \(\displaystyle{\frac 12 \brack \frac 12 -n}\) 的分母。

我们知道,

\[{x\brack x-n}=\sum_k\left\langle\left\langle\begin{matrix}n\\k\end{matrix}\right\rangle\right\rangle\binom{x+k}{2n} \]

而第 5 章第 72 题说,\(\binom{m/n}{k}\) 的分母是 \(n^{2k-v(k)}\)\(v(k)=\texttt{__builtin_popcount(k)}\)

\(\left\langle\left\langle\begin{matrix}n\\k\end{matrix}\right\rangle\right\rangle\)\(k=0\) 时等于 \(1\),大于 \(0\) 时是偶数。所以,原式的分母是 \(2^{4n-v(n)}\)

65.证明:

\[\int_0^1\int_0^1\dots \int_0^1f(\lfloor \sum_{i=1}^n x_i\rfloor)dx_1dx_2\dots dx_n=\sum_k\left\langle\begin{matrix}n\\k\end{matrix}\right\rangle\frac{f(k)}{n!} \]

我们先考察

\[\int_0^1\int_0^1\dots \int_0^1dx_1dx_2\dots dx_n=\int _0^n v(x)dx \]

\(v(x)\)

不妨转概率,其等于 \(n\) 个均匀分布于 \([0,1]\) 之间的随机变量的和在 \(x\) 上的概率密度。

根据 Irwin-Hall 分布,有:

\[v(x)=\sum_{k=0}^{\lfloor x\rfloor}\binom nk(-1)^k\frac{(x-k)^{n-1}}{(n-1)!} \]

原式左式即

\[\int_0^n\sum_{k=0}^{\lfloor x\rfloor}\binom nk(-1)^k\frac{(x-k)^{n-1}}{(n-1)!}f(\lfloor x\rfloor)dx\\ =\sum_{i=0}^{n-1}\int _i^{i+1}f(i)\sum_{k=0}^{i}\binom nk(-1)^k\frac{(x-k)^{n-1}}{(n-1)!}dx\\ =\sum_{i=0}^{n-1}f(i)\sum_{k=0}^{i}\binom nk(-1)^k\frac{1}{(n-1)!}\int _i^{i+1}(x-k)^{n-1}dx\\ =\sum_{i=0}^{n-1}f(i)\sum_{k=0}^{i}\binom nk(-1)^k\frac{(i+1-k)^n-(i-k)^n}{n!}\\ =\sum_{i=0}^{n-1}f(i)\left(\sum_{k=0}^{i}\binom nk(-1)^k\frac{(i+1-k)^n}{n!}-\sum_{k=1}^{i+1}\binom n{k-1}(-1)^{k-1}\frac{(i+1-k)^n}{n!}\right)\\ =\sum_{i=0}^{n-1}f(i)\left(\sum_{k=0}^{i}\binom nk(-1)^k\frac{(i+1-k)^n}{n!}+\sum_{k=1}^{i}\binom n{k-1}(-1)^{k}\frac{(i+1-k)^n}{n!}\right)\\ =\sum_{i=0}^{n-1}f(i)\sum_{k=0}^{i}\binom {n+1}k(-1)^k\frac{(i+1-k)^n}{n!}\\ =\sum_{i=0}^{n-1}f(i)\frac{1}{n!}\left\langle\begin{matrix}n\\i\end{matrix}\right\rangle\\ =\sum_{i=0}^{n-1}\left\langle\begin{matrix}n\\i\end{matrix}\right\rangle\frac{f(i)}{n!} \]

十分暴力,但好处是不需要智慧(除了转概率)。

具体数学使用了更组合意义的方法。仍然是转概率,但是后面是类似于欧拉数通项的那个组合意义证明:考虑 \(x_{1:n}\)\(n\) 个均匀分布于 \([0,1]\) 之间的随机变量。\(y\) 是其前缀和 \(\bmod 1\),而 \(\lfloor x_1+x_2+\dots+x_n\rfloor\) 就是 \(y\) 的下降个数!比较智慧。

69.求

\[\sum k^2H_{n+k}\delta k \]

首先显然可以变成

\[\sum x^{k}H_x\delta x,k=1,2 \]

对其分部求和即可。

70.证明前面(22)的复调和数

\[H_z=\sum_{n\ge 2}(-1)^n\zeta(n)z^{n-1} \]

\(\zeta\) 写成求和,随便写一下即可。

  1. (前面似乎是让你欧拉无穷乘积 \(\Rightarrow\) 魏氏无穷乘积?)然后求 \(\Gamma\) 导数。这个对数一下就可以了。
posted @ 2024-01-09 23:29  British_Union  阅读(82)  评论(0编辑  收藏  举报